Minor edits
This commit is contained in:
parent
9a531509ca
commit
89425270b9
@ -117,7 +117,7 @@ Show that $\mathcal{E}(\mathcal{A} + \mathcal{B}) = \mathcal{E}(\mathcal{A}) + \
|
||||
|
||||
\definition{}
|
||||
Let $A$ and $B$ be events on a sample space $\Omega$. \par
|
||||
We say that $A$ and $B$ are \textit{independent} if $\mathcal{P}(A \cap B) = \mathcal{P}(A) + \mathcal{P}(B)$. \par
|
||||
We say that $A$ and $B$ are \textit{independent} if $\mathcal{P}(A \cap B) = \mathcal{P}(A) \times \mathcal{P}(B)$. \par
|
||||
Intuitively, events $A$ and $B$ are independent if the outcome of one does not affect the other.
|
||||
|
||||
\definition{}
|
||||
|
@ -11,42 +11,45 @@ tosses of this die contain exactly one six? \par
|
||||
\hint{Start with small $l$.}
|
||||
|
||||
\begin{solution}
|
||||
$\mathcal{P}(\text{last } l \text{ tosses have exactly one 6}) = (\nicefrac{1}{6})(\nicefrac{5}{6})^l \times l$
|
||||
$\mathcal{P}(\text{last } l \text{ tosses have exactly one 6}) = (\nicefrac{1}{6})(\nicefrac{5}{6})^{l-1} \times l$
|
||||
\end{solution}
|
||||
|
||||
\vfill
|
||||
|
||||
\problem{}
|
||||
For what value of $l$ is the probability in \ref{lastl} maximal? \par
|
||||
The following table may help.
|
||||
The following table may help. \par
|
||||
\note{We only care about integer values of $l$.}
|
||||
|
||||
\begin{center}
|
||||
\begin{tabular}{|| c | c | c ||}
|
||||
\hline
|
||||
\rule{0pt}{3.5mm} % Bonus height for exponent
|
||||
$l$ & $(\nicefrac{5}{6})^l$ & $(\nicefrac{1}{6})(\nicefrac{5}{6})^l$ \\
|
||||
$l$ & $(\nicefrac{5}{6})^l$ & $(\nicefrac{1}{6})(\nicefrac{5}{6})^{l}$ \\
|
||||
\hline\hline
|
||||
1 & 0.83 & 0.133 \\
|
||||
0 & 1.00 & 0.167 \\
|
||||
\hline
|
||||
2 & 0.69 & 0.115 \\
|
||||
1 & 0.83 & 0.139 \\
|
||||
\hline
|
||||
3 & 0.57 & 0.095 \\
|
||||
2 & 0.69 & 0.116 \\
|
||||
\hline
|
||||
4 & 0.48 & 0.089 \\
|
||||
3 & 0.58 & 0.096 \\
|
||||
\hline
|
||||
4 & 0.48 & 0.080 \\
|
||||
\hline
|
||||
5 & 0.40 & 0.067 \\
|
||||
\hline
|
||||
6 & 0.33 & 0.055 \\
|
||||
6 & 0.33 & 0.056 \\
|
||||
\hline
|
||||
7 & 0.27 & 0.045 \\
|
||||
7 & 0.28 & 0.047 \\
|
||||
\hline
|
||||
8 & 0.23 & 0.038 \\
|
||||
8 & 0.23 & 0.039 \\
|
||||
\hline
|
||||
\end{tabular}
|
||||
\end{center}
|
||||
|
||||
\begin{solution}
|
||||
$(\nicefrac{1}{6})(\nicefrac{5}{6})^l \times l$ is maximal at $x = 5.48$, so $l = 5$. \par
|
||||
$(\nicefrac{1}{6})(\nicefrac{5}{6})^{l-1} \times l$ is maximal at $l = 5.48$, so $l = 5$. \par
|
||||
$l = 6$ is close enough.
|
||||
\end{solution}
|
||||
|
||||
|
@ -69,7 +69,7 @@ Come up with a strategy that produces better odds.
|
||||
|
||||
The remark from the previous solution still holds: \par
|
||||
When we're looking at the first applicant, we have no information; \par
|
||||
when we're looking at the second, we have no choices.
|
||||
when we're looking at the last, we have no choices.
|
||||
|
||||
\vspace{2mm}
|
||||
|
||||
@ -178,7 +178,8 @@ if we use the \say{look-then-leap} strategy detailed above? \par
|
||||
|
||||
\vspace{2mm}
|
||||
|
||||
Unraveling our previous logic, we find that the probability we are interested in is also $\frac{k-1}{x-1}$.
|
||||
Unraveling our previous logic, we find that the probability we are interested in is also $\frac{k-1}{x-1}$. \par
|
||||
\note{Assuming that $x \geq k$. Of course, this probability is 0 otherwise.}
|
||||
\end{solution}
|
||||
|
||||
\vfill
|
||||
@ -229,7 +230,7 @@ Let $r = \frac{k-1}{n}$, the fraction of applicants we reject. Show that
|
||||
\vfill
|
||||
|
||||
\problem{}
|
||||
With a bit of faily unpleasant calculus, we can show that the following is true for large $n$:
|
||||
With a bit of fairly unpleasant calculus, we can show that the following is true for large $n$:
|
||||
\begin{equation*}
|
||||
\sum_{x=k}^{n}\frac{1}{x-1}
|
||||
~\approx~ \text{ln}\Bigl(\frac{n}{k}\Bigr)
|
||||
|
@ -88,7 +88,6 @@ Given some $y$, what is the probability that all five $\mathcal{X}_i$ are smalle
|
||||
Say we have a random variable $\mathcal{X}$ which we observe $n$ times. \note{(for example, we repeatedly roll a die)}
|
||||
We'll arrange these observations in increasing order, labeled $x_1 < x_2 < ... < x_n$. \par
|
||||
Under this definition, $x_i$ is called the \textit{$i^\text{th}$ order statistic}---the $i^\text{th}$ smallest sample of $\mathcal{X}$.
|
||||
a
|
||||
|
||||
\problem{}<ostatone>
|
||||
Say we have a random variable $\mathcal{X}$ uniformly distributed on $[0, 1]$, of which we take $5$ observations. \par
|
||||
|
@ -1,81 +0,0 @@
|
||||
\section{The Secretary, Again}
|
||||
|
||||
Now, let's solve the secretary problem as as a stopping rule problem. \par
|
||||
The first thing we need to do is re-write it into the form we discussed in the previous section. \par
|
||||
Namely, we need...
|
||||
\begin{itemize}
|
||||
\item A sequence of random variables $\mathcal{X}_1, \mathcal{X}_2, ..., \mathcal{X}_t$
|
||||
\item A sequence of reward functions $y_0, y_1(\sigma_1), ..., y_t(\sigma_t)$.
|
||||
\end{itemize}
|
||||
|
||||
\vspace{2mm}
|
||||
|
||||
For convenience, I've summarized the secretary problem below:
|
||||
\begin{itemize}
|
||||
\item We have exactly one position to fill, and we must fill it with one of $n$ applicants.
|
||||
\item These $n$ applicants, if put together, can be ranked unambiguously from \say{best} to \say{worst}.
|
||||
\item We interview applicants in a random order, one at a time.
|
||||
\item After each interview, we reject the applicant and move on, \par
|
||||
or select the applicant and end the process.
|
||||
\item We cannot return to an applicant we've rejected.
|
||||
\item Our goal is to select the \textit{overall best} applicant.
|
||||
\end{itemize}
|
||||
|
||||
\definition{}
|
||||
First, we'll define a sequence of $\mathcal{X}_i$ that fits this problem. \par
|
||||
Each $\mathcal{X}_i$ will gives us the \textit{relative rank} of each applicant. \par
|
||||
For example, if $\mathcal{X}_i = 1$, the $i^\text{th}$ applicant is the best of the first $i$. \par
|
||||
If $\mathcal{X}_i = 3$, two applicants better than $i$ came before $i$.
|
||||
|
||||
\problem{}
|
||||
What values can $\mathcal{X}_1$ take, and what are their probabilities? \par
|
||||
How about $\mathcal{X}_2$, $\mathcal{X}_3$, and $\mathcal{X}_4$?
|
||||
|
||||
\vfill
|
||||
|
||||
\remark{}
|
||||
Now we need to define $y_n(\sigma_n)$. Intuitively, it may make sense to set $y_n = 1$ if the $n^\text{th}$
|
||||
applicant is the best, and $y_n = 0$ otherwise---but this doesn't work.
|
||||
|
||||
\vspace{2mm}
|
||||
|
||||
As defined in the previous section, $y_n$ can only depend on $\sigma_n = [x_1, x_2, ..., x_n]$, the previous $n$ observations.
|
||||
We cannot define $y_n$ as specified above because, having seen $\sigma_n$, we \textit{cannot} know whether or not the $n^\text{th}$
|
||||
applicant is the best.
|
||||
|
||||
\vspace{2mm}
|
||||
|
||||
To work around this, we'll define our reward for selecting the $n^\text{th}$ applicant as the \textit{probability}
|
||||
that this applicant is the best.
|
||||
|
||||
\problem{}
|
||||
Define $y_n$.
|
||||
|
||||
\begin{solution}
|
||||
\begin{itemize}
|
||||
\item An applicant should only be selected if $\mathcal{X}_i = 1$
|
||||
\item if we accept an the $j^\text{th}$ applicant, the probability we select the absolute best is equal to \par
|
||||
the probability that the best of the first $j$ candidates is the best overall. \par
|
||||
|
||||
\vspace{1mm}
|
||||
|
||||
This is just the probability that the best candidate overall appears among the first $j$, \par
|
||||
and is thus $\nicefrac{j}{n}$.
|
||||
\end{itemize}
|
||||
|
||||
So,
|
||||
\begin{equation*}
|
||||
y_j(\sigma_j) =
|
||||
\begin{cases}
|
||||
\nicefrac{j}{n} & x_j = 1 \\
|
||||
0 & \text{otherwise}
|
||||
\end{cases}
|
||||
\end{equation*}
|
||||
|
||||
\vspace{2mm}
|
||||
Note that $y_0 = 0$, and that $y_n$ depends only on $x_n$.
|
||||
|
||||
\end{solution}
|
||||
|
||||
\vfill
|
||||
\pagebreak
|
Loading…
x
Reference in New Issue
Block a user